Question 9 of 10 What is the slope of the line shown below? A. 4 B.-1/4 C.1/4 D.-4​

Question 9 Of 10 What Is The Slope Of The Line Shown Below? A. 4 B.-1/4 C.1/4 D.-4

Answers

Answer 1

Answer:

c = 1/4

-1-1 = -2

-3-5 = -8  

Step-by-step explanation:


Related Questions

A rectangular sheet of paper is 12 1/2 cm long and 10 2/3 cm wide.Find it's perimeter​

Answers

It is about 44cms for the perimeter.

.
Serenity has a points card for a movie theater.
She receives 80 rewards points just for signing up.
She earns 11.5 points for each visit to the movie theater.
She needs 195 points for a free movie ticket.
.
.
Write and solve an equation which can be used to determine v, the number of visits
Serenity must make to earn a free movie ticket.

Answers

11.5v + 80 = 195

11.5v - 80 = 195 - 80

11.5v = 115

divide each side by 11.5 and you should get v=10 (10 visits )

please help asap! ----------------------------

Answers

Answer:

[tex]f^{-1}(f(58))=58[/tex]

[tex]f(f(5))=11[/tex]

Step-by-step explanation:

We are given that the table which shows some inputs and outputs of the invertible function f with domain all real numbers.

We are given that

x         f(x)

5         9

3         -2

1          -5

18          -1

0           1

9           11

We have to find

[tex]f^{-1}(f(58))[/tex] [tex]f(f(5))[/tex]

We know that

[tex]f^{-1}(f(x))=x[/tex]

Using the property

[tex]f^{-1}(f(58))=58[/tex]

[tex]f(5)=9[/tex]

[tex]f(f(5))=f(9)[/tex]

[tex]f(f(5))=11[/tex]

Consider the equation 6x +7=3x � 5. Which of the following possible first steps would prevent having to deal with fractions when solving the equation?

Answers

Answer:

D.  I or II only

Step-by-step explanation:

By a small online search, I've found that the equation is:

6x + 7 = 3x - 5

And the options are:

I. Combining the 6x and 3x terms

II. Combining the 7 and 5

III. Dividing both sides of the equation  by 6

A.  I only

B.  II only

C.  III only

D.  I or II only

E.   I or II or III

So, let's solve the equation in such a way that we can prevent the use of fractions:

6x + 7 = 3x - 5

We can use I and II, combining one in each side, so we get (so we use I and II at the same time)

6x - 3x = -5 - 7

solving these, we get:

(6 - 3)*x = -12

3*x = -12

and -12 is divisible by 3, so if we divide in both sides by 3, we get:

x = -12/3 = -4

x = -4

So we avoided working with fractions, and we used I and II.

Then the first step could be either I or II (the order does not matter)

Then the correct option is:

D.  I or II only

Which of these numbers had exactly tow factors?

Select your answer. 7 8 9 10
A B C D



Answers

Answer:

prime numbers only have two factors...

7 is the only prime in the list

Step-by-step explanation:

Which represents f(x)=g
HURRRRRRY ITS A TIME TESSS

Answers

Answer:

B

Step-by-step explanation:

f(x) = g(x) at the intersection points of the two graphs.

The graphs intersect at x = -4 and x = 0.

At x = -4, f(-4) = g(-4).

At x = 0, f(0) = g(0)

Answer: B

Which equation in slope-intercept form represents a line that passes through the point (5, -1) and is
parallel to the lin y = 2x - 7?

A) y=-11x-7

B) y= -1/2x-7

C) y= 2x-9

D) y= -9x-7

E) y= 2x-11

Answers

Answer:

E) y= 2x-11

Step-by-step explanation:

as we know the equation of line parallel to

y =2x-7 or, 2x-y-7=0 is 2x-y+k=0

as the line passes through the point (5,-1)

substituting the value of x and y in equation 2x-y+k=0

we get,

2×5-(-1)+k=0

or, 10+1+k=0

or, k=-11

hence putting the value of k in ep 2x-y+k=0

we get 2x-y-11=0

or, y=2x-11

Question 1 Tell whether this table represent a proportional relationship. Give the constant proportionality.
O yes, k= 2/9
yes, k= 9
O not proportional

Answers

Answer:

yes , k = 9

Step-by-step explanation:

The equation of a proportional relationship is

y = kx ← k is the constant of proportion

To find k divide both sides by x

[tex]\frac{y}{x}[/tex] = k

Check each ordered pair in the table

k = [tex]\frac{18}{2}[/tex] = 9

k = [tex]\frac{45}{5}[/tex] = 9

k = [tex]\frac{63}{7}[/tex] = 9

Thus the relationship is proportional with k = 9

PLEASE HELP 20 POINTS + BRAINLIEST IF ANSWERED Which method correctly solves the equation using the distributive property? Negative 0.2 (x minus 4) = negative 1.7 Negative 0.2 (x minus 4) = negative 1.7. Negative 0.2 x minus 4 = negative 1.7. Negative 0.2 x = 2.3. x = negative 11.5. Negative 0.2 (x minus 4) = negative 1.7. x minus 4 = 0.34. x = 4.34. Negative 0.2 (x minus 4) = negative 1.7. Negative 0.2 x + 0.8 = negative 1.7. Negative 0.2 x = negative 2.5. x = 12.5. Negative 0.2 (x minus 4) = negative 1.7. Negative 0.2 x minus 0.8 = negative 1.7. Negative 0.2 x = negative 0.9. x = 4.5.

Answers

Step-by-step explanation:

oh oh can you capture a picture of it because you don't understand

HELP ASAP 10 POINTS AND BRAINLIST HURRY AND WILL GIVE 5 STAR AND A THANKS

Answers

A= 18 units squared
A of square = 9 units squared
A of triangles = 4.5 each unites squared

7) Qual número deve ser acrescentado a expressão 16x² - 2x, para que se obtenha um trinômio quadrado perfeito? * 5 pontos 1/3 1/2 1/4 1/5

Answers

Respuesta:

1

Explicación paso a paso:

Para completar el cuadrado tendremos que sumar la mitad del cuadrado del coeficiente de x a la expresión;

Coeficiente de x = -2

Mitad del coeficiente = (-2/2) = -1

Tomando el cuadrado del resultado = (-1) ^ 2 = 1

Por lo tanto, el número que se debe sumar para obtener un cuadrado perfecto es 1.

Solve for x. Round to the nearest tenth, if necessary.

Answers

Answer:

x ≈ 8.8

Step-by-step explanation:

Using the sine ratio in the right triangle

sin20° = [tex]\frac{opposite}{hypotenuse}[/tex] = [tex]\frac{PQ}{OQ}[/tex] = [tex]\frac{3}{x}[/tex] ( multiply both sides by x )

x × sin20° = 3 ( divide both sides by sin20° )

x = [tex]\frac{3}{sin20}[/tex] ≈ 8.8 ( to the nearest tenth )

What is the value of x?

Enter your answer, as a decimal, in the box.

Answers

Answer:

x = 50.6

Step-by-step explanation:

Choices:

One-sixth cups
Five-sixths cups
1 and two-thirds cups
1 and 5 / 6 cups

Answers

Answer:

Patel will require more orange juice = [tex]\frac{371}{660}[/tex] cups

Step-by-step explanation:

Patel needs the orange juice for his family = 3 cups

He needs more orange juice = 3 cups - Sum of juice squeezed from different oranges

[tex]=3-(\frac{13}{15}+\frac{1}{5}+\frac{9}{20}+\frac{5}{11}+\frac{7}{15})[/tex]

L.C.M. of the denominators = 660

[tex]=3-\frac{572+132+297+300+308}{660}[/tex]

[tex]=3-\frac{1609}{660}[/tex]

= [tex]\frac{1980-1609}{660}[/tex]

= [tex]\frac{371}{660}[/tex]

Therefore, amount of orange juice required more = [tex]\frac{371}{660}[/tex] cups

What is best buy £3.99 5kg or 3.2kg £2. 60?​

Answers

Answer:

I would say 5 kg because you get more kg with a little bit more money added

Please help me with this

Answers

Answer:

120 in^2

Hope it helps

PLESE ANSWE ASAPPPPP

Answers

Answer:

Number of terms: 2

Degree: 1

Step-by-step explanation:

✔️A term can either be a coefficient with a variable, a variable, or a constant.

In the polynomial given, 10y + 2, there are two terms:

First term is a coefficient with a variable = 10y

Second term is a constant = 2

The two terms are: 10y and 2

✔️Degree of a polynomial is the highest exponents possessed by any of its term.

10y has an exponent of 1.

The degree of the polynomial therefore will be 1

Given y = f(u) and u=g(x), find =f(g(x))g'(x) for the following functions.
dx
y = cos u, u = 4x - 3
dy
dx = f'(g(x))g'(x) = 0

Answers

Answer:

-4sin(4x-3)

Step-by-step explanation:

Given y = cos u, u = 4x - 3

dy/dx = dy/du * du/dx

dy/du = -sinu

du/dx = 4

dy/dx = -4sinu

since u = 4x - 3

dy/dx = -4sin(4x-3)

In In 5x + In In (x - 1) = 2

Answers

Answer:

exact form: x=-1/2

decimal form: x=-0.5

From the top of a building, 40 feet above the ground, a construction worker locates a rock at a 12° angle of depression. How far is the rock from the building? ​

Answers

Answer:

188 m

Step-by-step explanation:

[tex] \tan(12) = \frac{40}{x} \\ x = \frac{40}{ \tan(12) } \\ x = 188[/tex]

Answer:

I think that the other answer is WRONG

tan(12) = [tex]\frac{x}{40}[/tex]

x = tan(12)*40 = 8.5

Step-by-step explanation:

[tex]\frac{sin\left(78\right)}{40}=\frac{sin\left(12\right)}{x}[/tex] = 8.5

2 Select the correct answer. Which expression has a value closest to 1? (Use 3.14 as the value of 1.) OA 7 V10 B. V75-174 Oc. V37-26 3/5 - 20 V2 D.​

Answers

Answer:

It should be option c, [tex]\sqrt 37 - \sqrt26\\\\[/tex]

Step-by-step explanation: i hope this helps

foiuhgfdknsmlaeszxcyfvugbihnoj

Answers

Bufjfjfmfmgmbbbjbjbubbubububububehiveg veto veto so 5+2=7

Answer:

Given : A=(1,2)≡(x  

1 ,y  

1

) and B=(3,−2)≡(x  

2

,y  

2

)

Let M=(x,y) be the midpoint of AB

x=(  

2

x  

1

+x  

2

 

) and  y=(  

2

y  

1

+y  

2

 

)

Then, x=  

2

1+3

and y=  

2

2−2

 

⟹x=2 and y=0

∴M=(2,0)

∴ Coordinates of the midpoint of AB is (2,0)

Step-by-step explanation:

Select the correct answer. The product of two numbers is 21. If the first number is -3, which equation represents this situation and what is the second number? A. The equation that represents this situation is x − 3 = 21. The second number is 24. B. The equation that represents this situation is 3x = 21. The second number is 7. C. The equation that represents this situation is -3x = 21. The second number is -7. D. The equation that represents this situation is -3 + x = 21. The second number is 18.

Answers

Answer:

C

Step-by-step explanation:

Since it's the product, the two numbers must be multiplied together. -3 * x will equal 21, so -3x=21. By dividing both sides by -3, we see that x=-7

If given that the points on a line are L (8,4) and M (10. 14) and R is its midpoint on this, where is R on that line?​

Answers

Answer:

Step-by-step explanation:

The Midpoint formula is

[tex]M=(\frac{x_1+x_2}{2},\frac{y_1+y_2}{2})[/tex] where all of those x's and y's come from the coordinates of points L and M. Filling in:

[tex]M=(\frac{8+10}{2},\frac{4+14}{2}) =(\frac{18}{2}, \frac{18}{2})=(9,9)[/tex]

identify the maximum and minimum values of the function y=10cosx in the interval [-2pie, 2pie]. Use your understanding of transformations, not your graphing calculator.

Answers

Answer:

3 x + 2 y + z/ x + y + z ,  x = 2 ,  y = 3 ,  z = 1

tan ( x ) ,  x = − π

cot ( 3 x ) ,  x = 2 π /3

Step-by-step explanation:

Estimate the number of square yards of carpeting needed to cover a floor 10'3" by 15'9.

Answers

Answer:

17.9375 square yards

Step-by-step explanation:

Let us have a common unit

What we have here is the case of inches and ft

10 ft 3 inches

1 ft = 12 inches

so 3 inches is 3/12 = 0.25 ft

= 10+0.25 = 10.25 ft

15 ft 9 in

= 15 + 9/12 = 0.75 + 15 = 15.75 ft

So let us convert to yards ;

Mathematically, 3 ft = 1 yard

so 10.25 ft = 10.25/3 = 3.4167 yards

15.75 ft = 15.75/3 =5.25 yards

So the square yards would be the product of this two;

which is;

(10.25/3) * (15.75/3) = 17.9375 square yards

yeaaaaaaaaaa help plzzzzzzzzzz

Answers

Answer:

See Explanation

Step-by-step explanation:

Given

The attached graph

Required

Point on the line

The options are not given. So, I will provide general answers.

To do this, we simply write out the coordinates that lie on the line.

Some of them are:

[tex](0.5,3)[/tex]

[tex](1.25,8)[/tex]

[tex](1.5,10)[/tex]

Solve for x. Round to the nearest tenth of a degree, if necessary

Answers

Answer is x=50.1

Step 1

Tanx=opposite/adjacent

Step2

Tanx=6.1/5.1

x=arctan(6.1/5.1)

x=50.1

Answer in degrees as required

How do I answer number 1

Answers

Answer:

Haley simplified correctly

Step-by-step explanation:

According to indices rule, if we have the same base and we are to multiply two numbers raised to exponent, we are to add the powers on the bases

So, it is expected that;

x^3 * x^2 = x*(3 + 2) = x^5

However, x^3 + x^2 stays the same (but of course can only be factorised)

So we can conclude that only Haley’s answer is correct

someone please help me ASAP!

Answers

Answer:

214°

Step-by-step explanation:

The measure of an arc that sees the center angle of the circle is equal to the very same angle that it sees

Since the measure of circle is 360° and arc AB is given as 146° the measure of arc ACB should be 360 - 146 = 214°

Other Questions
Raphael's mural The School of Athens depicts a philosophical discussion. Somewriters have noticed that there are no women depicted in the work. Therewere women active in ancient Greece in the realms of poetry and literature. Does that make this a sexist painting? Arvin has $10000 to invest. He invests part in a term deposit paying 5%/year, and the remainder in Canada savings bonds paying 3.4%/year. At the end of the year, he earned simpler interest of $413. How much did he invest at 5%/year? An IUDO offers long-term protection from pregnancy.is unlikely to cause complications.contains hormones.O causes a long delay in the return of fertility. Find the distance between A (2,0,-1) and B (3,1,4) and find the mid-point of line segment AB." Question 2 of 10Look at this painting by Rubens. It features what ideal of beauty for women ofthe time?O A. Passionate and chasteO B. Pale and distantO C. Curvy and voluptuousO D. Tall and dignifiedSUBMIT which city is described in the first paragraph? Number of RepresentativesState196019701980Colorado456New York413934Pennsylvania272523Texas232427What was one result of the changes shown in the table?O A. A rise in wages throughout the nation's industrial areasO B. Decreased federal spending on antipoverty programsOC. A decline in support for agricultural subsidiesO D. Increased political power for RepublicansSUBMIT Q011) The Doppler effect a. occurs when the frequency of sound waves received is lower if the wave source is moving toward you than if it's moving away. b. occurs when the pitch of a sound gets lower if the source is receding. c. is the basic explanation for the blue shift of light in our Universe. d. can be applied only to sound waves. discuss cybersecurity in today's current situation Difference between DIRECT and INDIRECT ratio? Compare the rates for different numbers of texts. If Roger's father wants to get a 600-text message plan, what is the difference in price for the Dial It Up and Ring Ring plans? f(X) = 10x^3 find inverse solve the following simultaneous linear equations by the substitution method. 3x + 4y = 12x + 3y = 1 Thomas McDonald earns $12,550 every month and sets aside the minimum amount for saving. How much would Thomas McDonaldbe saving from his income?1. $677.52. $1,2553 .$2,5104. $3,765 do thyroid hormones influence heart rate from influencing the heart itself, or the attached to it? For this graph, mark the statements that are true.A. The domain is the set of all realnumbers.B. The range is the set of all realnumbers greater than or equal tozero.C. The domain is the set of all realnumbers greater than or equal tozero.D. The range is the set of all real numbers. (Will put as Brainliest) Please help! :")A speciality candy shop makes chocolate covered cherry and graham cookie bites. The cherry is spherical with a diameter of 3cm and the graham cookie is a rectangular prism with a base measuring 5cm by 5cm and a thickness of 0.5cm. What is the total surface area of the bite if each piece is drenched in chocolate from top to bottom before being put together, to the nearest tenth of a square centimetre. define gender identity on Its ImpactHow might the author convey the same information using a chronologicalstructure?A. By putting the tips in order of when they are mostly likely to berelevant in one's dayB. By comparing conscientious behavior to full-on activism in termsof their benefitsC. By showing the environmental consequences of failing to performeach actionD. By arranging the actions based on their ranking, starting with themost "Green Earths"SUBMIT Use the elimination method to solve the system of equations. A. Infinitely many solutions B. (10, 10)C. (16, 18)D. No solution